LSAT and Law School Admissions Forum

Get expert LSAT preparation and law school admissions advice from PowerScore Test Preparation.

 Laura Carrier
PowerScore Staff
  • PowerScore Staff
  • Posts: 38
  • Joined: Oct 04, 2015
|
#20231
Hi Sherry001,

Your question about answer choice (C) is very interesting! The stimulus essentially uses the growing economic incentive created by an expanding human population and a diminishing amount of unoccupied space on Earth as an implicit sufficient condition, the presence of which we can regard as sufficient to override any existing disincentives (e.g., high cost) to using the technology to build moon colonies, thus leading to the conclusion that the colonies will almost certainly be built. So you can think of the conditional content of the argument roughly as follows:

If the economic incentive grows, then the colonies will be built.

But it’s also a bit of a hybrid argument, since there is also the causal implication that the growing economic incentive will almost certainly cause the disincentive to be overridden and the colonies to be built.

Notice how answer choice (A) very cleverly addresses both of these features of the stimulus by describing the flaw as taking for granted that the economic incentive will grow sufficiently (become sufficient) to cause (act as a causal agent) the colonies to be built (necessary effect).

Answer choice (C) is tricky because, as you noticed, it also picks up on the fact that the stimulus has implied a quasi-sufficient-necessary relationship (with a hint of a causal relationship), but in fact this answer choice mistakenly reverses the implied conditional relationship in the stimulus, by portraying the economic incentive as a necessary condition to the colony-building rather than the sufficient condition that the stimulus made it.

Answer choice (C) could have worked if the stimulus had implicitly reasoned that, “only if the economic incentive grows, will the moon colonies be built.” Then we could have agreed with (C) that the stimulus was mistaken to reason something like, “if colonies, then economic incentive,” if there is actually a possibility that colonies might be built without the economic incentive--i.e., that the argument's sufficient condition might actually have existed without the presence of the necessary condition.

But since the stimulus actually says something more like, “if incentive, then colonies,” it is not pointing out a flaw for (C) to say that the colonies (the necessary condition) might be present without the incentive (the sufficient condition), because the presence of the necessary condition never tells us that the sufficient condition must be present. Even if the colonies might exist without the incentive (as (C) suggests), this would not make the stimulus wrong to say, “if there is an incentive, then there will almost certainly be colonies.” Calling this a flaw makes (C) essentially guilty of a mistaken reversal.

I hope this clarifies things!
Laura
 srcline@noctrl.edu
  • Posts: 243
  • Joined: Oct 16, 2015
|
#24498
Hello,

My issue with answer choice A is the word "sufficiently" I do not see conditional reasoning in this stimulus. There is casual reasoning, but I thought casual reasoning and conditional reasoning are two different concepts.

Thankyou
Sarah
 Nikki Siclunov
PowerScore Staff
  • PowerScore Staff
  • Posts: 1362
  • Joined: Aug 02, 2011
|
#24623
Hi Sarah,

I'd caution you against single-mindedly focusing on keywords (such as "sufficiently"), because when taken out of context, such a focus can lead you down the wrong path. You end up not seeing the forrest from the trees, as it were.

In the simplest terms possible, the author assumes that the benefits of the proposed course of action - building colonies on the moon - will outweigh the costs (which are, admittedly, quite high). Answer choice (A) provides the closest answer to this prephrase: the economic incentives will grow sufficiently (i.e. they will grow enough) to cause such a project to be undertaken. If you aren't fully convinced, approach answer choice (A) as if it were the answer to an Assumption question and try the Assumption Negation Technique: what if the project costs ultimately outweigh the benefits, and the economic incentives never become sufficiently strong to cause such a project to be undertaken? Clearly, then, the conclusion would make little sense. Therefore, the author must have taken for granted, i.e. assumed, that the economic incentives will indeed grow sufficiently to cause such a costly project to be undertaken.

Hope this helps!

Thanks,
 srcline@noctrl.edu
  • Posts: 243
  • Joined: Oct 16, 2015
|
#24956
Hello Nikki

Okay, you are right, I think I am absolutely getting hung up on the details here instead of seeing the big picture with these flaw questions. After looking at your response A is making sense. Would another issue with B would be that Max never says that building colonies on thee moon would "be the only way", this is not a flaw in his argument? Also if I get stuck on these types of questions can I use the assumption negation technique?

Thankyou for your explanations
Sarah
 Nikki Siclunov
PowerScore Staff
  • PowerScore Staff
  • Posts: 1362
  • Joined: Aug 02, 2011
|
#25018
Sarah,

You are correct. Flaw questions should ideally be prephrased; but if you can't do that, you can use your Assumption and Weaken question skill set in your line of attack:

1. When analyzing any answer to a Flaw question that begins with one of the following, you can use the Assumption Negation Technique to validate or disprove it:
The author presumes, without justification, that...
The author takes for granted that...
For the author to be assuming/taking for granted the claim in such answers, such a claim must technically be an assumption upon which the conclusion relies. Consequently, for that answer choice to be correct, negating the statement embedded in it should undermine the conclusion of the argument.

2. Also, keep in mind that arguments frequently fail to take into account any number of different possibilities, most of which are entirely irrelevant to the logical validity of the conclusion. A discrete number of possibilities, however, are relevant and should have been taken into account. Why? Because, if true, they could potentially weaken the argument. So, you can approach any answer that begins in one of the following ways as a potential Weaken answer:
The author overlooks/ignores the possibility that...
If you are able to say to yourself, “This would weaken the argument,” or, “This is what I would say to attack the author,” you already know an avenue that would hurt the argument, and you can use that to help understand the flaw.

Does that make sense? Let me know.

Thanks!
 ShannonOh22
  • Posts: 70
  • Joined: Aug 15, 2019
|
#68539
Very quick question here - when an answer choice reads "It takes for granted that"...how should we interpret that phrase? I thought something being "taken for granted" would mean it is unstated, and/or assumed?

The explanations provided for A and against B center on the fact that the author never states anything about colonizing the moon being the ONLY option. But wouldn't the fact that it is unstated be part of being "taken for granted"?

Should I be reading an answer choice that starts with "It takes for granted that" to mean "it mistakenly states that..." and the answer must still be found somewhere in the text of the stimulus, as it does with A?

I hope that's not too convoluted...I'm trying to get a better grasp on some of the phrases that seem to have different meanings in the real world versus the LSAT world.

Thank you!
User avatar
 KelseyWoods
PowerScore Staff
  • PowerScore Staff
  • Posts: 1079
  • Joined: Jun 26, 2013
|
#71163
Hi Shannon!

"Takes for granted" means the same thing as "assumes." It's not quite the same as being unstated...it's unstated AND necessary for the argument. The argument does not assume that colonizing the Moon is the ONLY option, just that it is an option that is almost certainly going to come to pass. The argument does however assume that the economic incentive is going to grow sufficiently to cause costly Moon colonization to happen. How do we know the author assumes this? Because the author jumps from the premise that the economic incentive is going to increase to the conclusion that Moon colonization is almost certainly going to happen.

Assumptions can be tricky--sometimes they seem like they're coming out of nowhere (often Defender Assumptions might look like this), sometimes they seem so obvious that we think the author already explicitly state them (often Supported Assumptions look like this). But assumptions are not explicitly stated, even if elements of them are. In this case, the author has not explicitly stated that the economic incentive is going to grow sufficiently, but has assumed that in jumping to the conclusion.

Hope this helps!

Best,
Kelsey
 chiickenx
  • Posts: 21
  • Joined: Apr 30, 2019
|
#74620
Hi, this is my explanation for why B is wrong. I was reading through the other explanations but couldnt make sense of it. Can somebody tell me if I am even remotely close to understanding why B is wrong? Thank you!

(B) is pretty good IF we knew that the earth will become severely overcrowded. If we do not assume as such, even supposing that the only way earth could be relieved of severe overcrowding is via moon colonies, its unclear if moon colonies will almost certainly be built. That's why (A) is just so much better because (A) is the assumption we need to even make (B) work in favor of this argument.
 Adam Tyson
PowerScore Staff
  • PowerScore Staff
  • Posts: 5153
  • Joined: Apr 14, 2011
|
#74638
I don't think I would go so far as to say that answer B is "pretty good," chiickenx. Even if we add in our own assumptions about overcrowding getting very severe, the author still doesn't need to assume that moon colonies are the only solution. Perhaps extreme population control measures, like forced sterilization of 90% of the population, or a "Logan's Run" approach of killing everyone over the age of 30, might also do the trick? The issue with answer B is that the author never needs to rely on moon colonies as the ONLY solution. It's just "the incentive to do it is growing, so eventually we will do it."

The only required assumption is that at some point the incentive will be strong enough to overcome all potential objections to doing it. It's growing, sure, but will it grow enough to make us decide to actually do it? For the past 4 years or so, I have felt a growing desire to leave the United States and move to a remote cabin in the Northwest Territories, never to return. Does that mean that I will definitely do that someday? Only if we assume that the reasons for leaving will reach a tipping point where they completely outweigh the reasons for staying where I am.

Answer B isn't "pretty good." In fact, it's downright awful. Don't help it! The right answer never needs any help!
 Sambenz
  • Posts: 15
  • Joined: Jun 03, 2020
|
#76082
I'm not sure I totally understood everything mentioned in this thread for why B is wrong, but this is why I understand A as the correct answer and why B is attractive:

This is the argument structure:

Premise 1: Humans have the technology to go to the moon.
Premise 2: Doing so would be costly
Premise 3: There is likely overcrowding issues in the future.
Premise 4: The overcrowding causes a growing economic incentive to relocate people
Conclusion: This growing economic incentive guarantees the Moon colonies will be built, and overcrowding relieved.

Clearly, the key flaw here is the causal relationship at the end:

Economic incentive :arrow: certain moon colonies.

But, if you don't read the question carefully you can misinterpret the relationship as:

Severe overpopulation :arrow: certainly moon colonies

Meaning, since we need to resolve the overpopulation crisis, that causes moon colonies to be built.

If you interpret the relationship that way, then B appears extremely attractive as it attacks the relationship that overcrowding causes the moon colonies. But that is not the relationship that the argument asserts, so B is incorrect.

I also felt that The final words "and severe overcrowding on Earth relieved" attempts to misdirect students as well. The economic incentive part is hidden in the stimulus between different concepts regarding overpopulation, to try to trick students. As well, there is this sci fi futuristic feeling that the moon is the solution to overpopulation that may also be at play here.

This is my interpretation.

Get the most out of your LSAT Prep Plus subscription.

Analyze and track your performance with our Testing and Analytics Package.